« first day (14 days earlier)      last day (560 days later) » 
00:00 - 14:0014:00 - 00:00

12:03 AM
Short question. Question contains please.
0
Q: Counterfeit Coin With 8 Coins

J. Andrews How many weighings of a balance are necessary to determine if a coin is counterfeit among eight coins. The counterfeit coin is either heavier or lighter than the other coins. I understand the reasoning behind this problem when you know how the weight of the counterfeit coin compares to the r...

Short title.
 
12:27 AM
0
Q: Formula for nth term of sequence?

Enigma99How do you find a formula for the nth term of the sequence defined by: $x_n$$_+1$=$x_n$+$2$x_n$$_-1$ Where $x_0$=4 and $x_1$=-1 ? n=1,2,3,...

Short question.
0
Q: Linear Algebra Vector Space

mathloverFrom textbook definition of vector spaces, we know that it is a set V together with an operation of addition, x+y for any x,y $\epsilon$ V, and an operation of scalar multiplication, sx for any x$\epsilon$R. This is straight-forward, but how would you know if x+y of x,y $\epsilon$ V, would actua...

Short title.
0
Q: Subgroups of a group with prime order.

John CenaQuestion: Suppose the order of a group $G$ is prime. Does $G$ have subgroups? If so how many? Attempt: Yes, G has subgroups. Let $G$ be a group of order $p$, where $p$ is prime. Let $H$ be a subgroup of $G$. By Lagrange's Theorem $\mid H \mid \vert \mid G \mid$. But since the only (positive) div...

Tagged proof-verification.
0
Q: Poincare Theorem

user290472The Poincare Theorem is, "If a limit cycle exists in the second order autonomous system \dot{x}=f(x), then N=S+1". Does this theorem gives only a necessary condition for existence of a limit cycle or it is also sufficient? If the condition is not sufficient, is there any counterexample? Thanks fo...

Short title.
0
Q: Formal definition of an array

onlycparraI need to know how to define formally an array of length n and composed by ones and zeros depending on certain condition Maybe something like this? $\vec{A}= (a_1,a_2,a_3...)\mbox{ with } a_i=1 \mbox{ where } X_i=Y\mbox{ or } a_i=0 \mbox{ where }X_i\neq Y\hspace{0.5cm}; 1 \le i \le n$

Welcome to Math.SE, onlycparra. Questions tend to get more attention when they have a tag for a broad area of mathematics relevant to the question. Some of these tags might fit. (autocomment)Normal Human 20 secs ago
 
12:54 AM
0
Q: Taylor's Theorem Question: finding the limit as x approaches 0

Minya YaoI am trying to calculate $$\lim\limits_{x \to 0} \frac{(x-\sin x)^{70}}{1-\cos x^{105}}$$ Here is my attempt: $ $ write $\cos$ and $\sin$ as Taylor series, and plug back into the original expression yields:$$\lim\limits_{x \to 0} \frac{(x^3/3!-x^5/5!+x^7/7!-...)^{70}}{x^{210}/2!-x^{420}/4!+x^{63...

Title contains question, 0.
0
Q: Jacobi iteration converges iff Gauss-Seidel iteration converges

Alon GelberLet $A ∈ R^{2×2}$. Show that the Jacobi iteration converges for all starting guesses if and only if the Gauss-Seidel iteration converges for all starting guesses.

Short question.
0
Q: Termwise integration of Fourier Series PROBLEM

LukeI need help with this problem, I keep solving it my own way but am not getting the same series as a result.

Title contains problem. Short question.
0
Q: Characterization of $l^{2}(\mathbb{N])$-sequences

user159517I'm curios whether or not the following implication is true: If $x_{n} \notin \ell^2{(\mathbb{N})}$, is there necessarily a sequence $y_{n} \in \ell^{2}(\mathbb{N})$ such that $x_{n}y_{n} \notin \ell^{2}(\mathbb{N})$?

Short title.
0
Q: Does ∑(-1)^n * (1+ 1/n)^n converge or diverge?

user290431Hey i have tried root and alternant but couldn't make it, how u guys solve this, if u r in an exam. which test should i tried? hel please thank you to everybody bye good night and god bless u.

Question contains please.
 
1:13 AM
0
Q: Order of a Permutation of s

GC10101A permutation $s$ of $n$ elements has order $2016$ ). What is the minimal value of $n$? Give an example of an $s$ for the smallest $n$. How do I solve such a problem? I know I must consider the cycle decomposition of $n$.

Short title.
0
Q: Discrepancy between tensor product and Kronecker product?

man_in_green_shirtI have tried evaluating $|\psi\rangle^2$, both using the tensor product notation and the Kronecker product. Why do the results look so different? Are they even right?

Short question.
0
Q: Optimal Selection of Random Variables with Constraints

Joseph ZambranoI am looking for some direction on the following problem. I am sure there are many variations with specific distributions, constraints, ect. but I will phrase it in a fairly general form. Suppose we have a set of $N$ real random variables $S=\{X_1, ..., X_N\}$ and we know (or can estimate with r...

Question contains please.
 
0
Q: Code snippet overflows container

Sexy TurnipThe code snippet in this question causes the viewport to expand wildly in iOS Safari (iOS 9.1) HTML horizontal image scroll not working

 
0
Q: Bounding the values of angles from their cosines relationship

SpiderRicoIf the ratio of cosines of 2 angles are bounded from above, can we conclude that the ratio of the angles are bounded as well ?

Short question.
0
Q: Summation with a variable as the upper limit

Jacob Copland$$\sum_{n=1}^m \frac{k \cdot k! \cdot \binom{m}{n}}{m^n} = ?$$ My attempts on the problem: I tried writing out the summation. $$1+\frac{2(m+1)}{n} + \frac{3(m-1)(m-2)}{m^2} + \cdots + \dfrac{m\cdot m!}{m^m}$$ I saw that the ratio between each of the terms is $\dfrac{\dfrac{n}{n-1} (m-n+1)}{m...

Question contains please.
0
Q: Does propositional resizing preserve truth of propositions in HoTT?

user287393According to Axiom 3.5.5 of the HoTT book, we have propositional resizing if there is a function f such that f : Prop$_u$$_{_i}$ $\rightarrow$ Prop$_u$$_{_{i+1}}$ isequiv(f) The idea is that p and (f p) are in some sense 'equivalent' for any p : Prop$_u$$_{_i}$, and similarly for q and...

0
Q: Functions of random variables

user290220Please consider Example 4.8 on http://www.probabilitycourse.com/chapter4/4_1_3_functions_continuous_var.php Would someone mind explaining how $P(X^2≤y)=P(−\sqrt{y}≤X≤\sqrt{y})$ and how $R_y=[0,1]$ (essentially if $-1<x<1$ how does it become $0<y<1$)? Thanks.

Short title. Question contains please.
Questions tend to get more attention when they have a tag for a broad area of mathematics relevant to the question. Some of these tags might fit. (autocomment)Normal Human 29 secs ago
0
Q: A Calculus math problem that I don't understand and need help answering

user290452A curve is given by the equation: 3(x+1)^2-9(y-1)^2=32 (A) Find the coordinates of the two points on the curve at which x = 3 (B) Find the slope of the curve at each of these two points.

Title contains help, problem.
1
Q: $S^n$ cannot be an $H$-space if $n$ is even, easy approach?

Daniel BlakeWhat is a comparatively elementary way to see that $S^n$ cannot be an $H$-space if $n$ is even that does not invoke, say, Adams's full solution to the Hopf invariant one problem?

Title contains easy. Short question.
 
1:42 AM
0
Q: Show that if $f:\mathcal{O}\rightarrow\mathbb{R}$ is not constant, then it fails to have any local maximizer and local minimizer

Simple For real numbers $a,b,c$ and $d$, define $\mathcal{O}=\{x|cx+d\neq 0\}$. Then define $$f(x)=\frac{ax+b}{cx+d}\;\;\;\text{for all $x\in\mathcal{O}$}$$Show that if $f:\mathcal{O}\rightarrow\mathbb{R}$ is not constant, then it fails to have any local maximizer or local minimizer. Sketch the graph...

Tagged proof-verification.
0
Q: Does there exist a set which is its own complement? If so can you give an example

elasolovaIs it possible that there exist a set U that has the property U=UC?

Short question.
0
Q: Sum of Convergent Series Estimate

BellamyIf we used the first 2015 terms to estimate the sum of the convergent series $\sum\limits_{i=1}^n$ $(-1)^(n+1)/(n^2)$ would it be an overestimate or an underestimate? I really have no idea where to start with this so could someone help me out please? P.S. I'm having a bit of trouble with the Ma...

Question contains please.
 
2:00 AM
0
Q: Prove the function is a homomorphism and calculate the image

MikaellaConsider the function f: (Z_10, +_10) --> (Z_20, +_20) defined by f(n)=2n. Show that f is a homomorphism and calculate the image. [NOTE: Z_m = {0,1,2,...,m-1} is a group under the operation +_m defined via i +_m j equals the remainder of i+j when divided by m.] I understand that to prove homom...

Welcome to Math.SE, Mikaella. This site uses MathJax formatting of formulas. More tips here. (autocomment)Normal Human 21 secs ago
0
Q: Showing Minkowski integral inequality with p = 2

clocktowerI have shown: $$\bigg(\int_{0}^{1}f(t)g(t)dt\bigg)^{2} \leq \int_{0}^{1}g(t)^{2}dt\int_{0}^{1}f(t)^{2}dt$$ and now I'd like to use this to show the Minkowski inequality for $p=2$, i.e. $$\Bigg(\int_{0}^{1}(f(t) + g(t))^{2}dt\Bigg)^{\frac{1}{2}} \leq \Bigg(\int_{0}^{1}f(t)^{2} dt\Bigg)^{\frac{...

Questions tend to get more attention when they have a tag for a broad area of mathematics relevant to the question. Some of these tags might fit. (autocomment)Normal Human 21 secs ago
 
0
Q: How has MSE changed your phone's keyboard input app?

Matt SamuelThis "sentence" (loosely speaking) was automatically generated by Swype on my android: The only possibility is considering isomorphism classes and programs that the absolute value of a Schubert polynomial formula for equivariant cohomology of my ears are congruent and not equal to the point. ...

 
2:18 AM
0
Q: $\bar {int(A)}$ for all closed sets $A \subseteq R$ is true?

casserolesingsIs "$\bar {int(A)}$ for all closed sets $A \subseteq R$" true? How can I prove it?

Short question.
0
Q: Calculus problem I also do not understand and please help me find a clear answer to it.

user290452Consider the closed curve in the xy plane given by x^2+2x+y^4+4y=5 (A) Show that dy/dx = - x+1/2(y^3+1) (B) Write an equation for the line tangent to the curve at the point (-2,1)

This site uses MathJax formatting of formulas. More tips here. (autocomment)Normal Human 21 secs ago
0
Q: Trouble in seeing that if two subsets are conjugates, they are of equal cardinality.

Henri LSuppose $H_1= g \cdot H_2 = gH_2g^{-1}$. Is there an easy way to see $|H_1|=|H_2|$? Thanks!

Title contains troubl. Short question. Tagged proof-verification.
0
Q: Unitary matrix eigenvalue

user3687267I need some hint to solve this problem. Let U be a unitary matrix. Prove that if λ is an eigenvalue of U, then |λ| = 1.

Short title. Short question.
0
Q: Trigonometric Application

Dunja ElezAt a seaport, the depth of the water $h$ metres at a time $t$ hours during a certain day is given by this formula; $$h=1.8sin[2\pi{t-4.00\over12.4}]+4.3$$ What is the maximum depth of the water? When does it occur? I know the maximum depth is 4.9 metres but what I don't know is how to solve the ...

Short title.
0
Q: Integration help needed Gamma

YaarCan anyone tell me how can I solve this?? $$\frac{\theta^n}{\Gamma(n)}\int_1^{\infty} \left(\frac{x-1}{x}\right)^{2n-2}x^{n-1}e^{-\theta x} $$ Where $\theta > 0$

Words such as help do not add information to titles. Please edit the title so that it better describes the specifics of your question. Do not hesitate to make it longer or include a formula if needed. More tips here. (autocomment)Normal Human 21 secs ago
0
Q: convergence of series (sqrt(n+1)+sqrt(n))/n^p

gabrielchuadiscuss the convergence of the series for all p. 2/n^(p-1/2)= 2*sqrt(n)/n^p < (sqrt(n+1)+sqrt(n))/n^p < 3sqrt(n)/n^p < 3/n^(p-1/2) so ~ (sqrt(n+1)+sqrt(n))/n^p ~ K/n^(p-1/2) converges when p>3/2, diverges when p<=3/2 Intuition: I there exist M, an Is my reasoning right?

Welcome to Math.SE, gabrielchua. This site uses MathJax formatting of formulas. More tips here. (autocomment)Normal Human 21 secs ago
0
Q: At least one flip is Heads

StephI have been stuck on this for some time I am not sure how to approach this type of problem: Let $n \ge 1$ be an integer, and for each $i=1,2,...n$ let $p_i$ be a real number such that $0<p_i<1$ $\sum_{i=1}^n p_i$$\prod_{j=i+1}^n (1-p_j)$$=1-\prod_{i=1}^n (1-p_i)$ Example: for $n=1$ the equat...

Short title.
 
2:54 AM
0
Q: If $|G| = 120$ then $G$ has a subgroup of index $3$ or $5$ (or both)

BungoThis is exercise 1.C.4 in Isaacs, Finite Group Theory. I think I have a proof, but would like to verify the proof and also inquire whether it can be shortened significantly. Let $|G| = 120 = 2^3 \cdot 3 \cdot 5$. Show that $G$ has a subgroup of index $3$ or a subgroup of index $5$ (or both). ...

Tagged proof-verification.
0
Q: Fermat's Little Theorem Induction Question

Tomas SmithLet $M = p_1 *p_2 * ...*p_n$ be a positive integer with prime factorization. Let $gcd(a,M) =1$. Prove that $a^ {(p_1 -1)*(p_2 -1)...*(p_n -1)} = 1 (mod M)$, by induction. The base case is just Fermat's Little Theorem but I am unsure how to proceed.

0
Q: On group and graph isomorphisms

ArulAre two finite groups isomorphic iff their subgroups are? Are two finite graphs isomorphic iff their subgraphs are?

0
Q: Random Variables, Die toss

user273323A fair die is tossed twice. Let $X$ = the ssm of the faces, $Y$= the maximum of the two faces, and $Z$=|face 1 - face 2|. write down the value of $X,Y,$ and $W=XZ$ for each outcome $w\in\ S$ I already found the value and range of $X,Y$ but I'm not sure how to find $W=XZ$. I saw someone post a s...

Short title.
Words such as question do not add information to titles. Please edit the title so that it better describes the specifics of your question. Do not hesitate to make it longer or include a formula if needed. More tips here. (autocomment)Normal Human 21 secs ago
 
3:12 AM
0
Q: Is the Galois group of the polynomial $x^3-3x+3$ trivial over $\mathbb{Q}(i\sqrt{15})$?

Sir JectiveI would appreciate it if you could verify my reasoning is correct or inform me of where the flaws in my reasoning are for the following problem: "Find the Galois Group of $x^3-3x+3$ over (a)$\mathbb{Q}$ (b)$\mathbb{Q}(i\sqrt{15})$ (a) Since the discriminant of $X^3-3x+3$ is negative, I know t...

Tagged proof-verification.
0
Q: Proof that sum of two inequalities (with same direction of the sign) holds for positive numbers

Eran MedanI am sure this is a duplicate, but sometimes a question is so basic and silly that it gets ignored / deleted by experts so a newbie to the community is missing basic information because "everyone knows that" (I think this is some famous paradox but this is another topic) The question is dead si...

Question contains please. Tagged proof-verification.
0
Q: In the symmetric group of degree 4, S_4. Find a 3-subgroup and a 4-subsgroup of Slow.

Mario Alejandro MartinezIn the symmetric group of degree 4, S_4. Find a 3-subgroup and a 4-subsgroup of Slow. Can anyone help me? Thanks for all your help!

Short question.
0
Q: Reference book for quadratic equations with hard exercises?

Angelo MarkI'm teaching advanced level mathematics. And I need good question banks on quadratic equations . More likely good question bank on advanced level maths.

Title contains hard, exercise. Short question.
0
Q: Suppose S and T are two disjoint and compact sets in a normed vector space W.

calidreamingSuppose S and T are two disjoint and compact sets in a normed vector space W. Then there are $s \in S$ and $t \in T$ s.t. the infimum of $||s-t||=||s-t||$? Is this true? How can I prove that?

Short question.
0
Q: Euler Transform elementary Proof

KeithIn this webpage Computing the Digits in π there is a proof of the Euler Transform. The proof there relies on measure theory and Lebesgue integration, I haven't studied that yet. In page 22 there is the following statement: Euler didn’t actually prove any general theorems about this transfo...

Title contains elementary.
0
Q: Radius of covergence $\sum_{k = 0}^{\infty}\frac{ k^{2 k + 5} \ln^{10} k \ln \ln k}{(k!)^2} x^k$

G HI want to find the radius of covergence of $$\sum_{k = 0}^{\infty}\frac{ k^{2 k + 5} \ln^{10} k \ln \ln k}{(k!)^2} x^k$$ Thank you for any help!

Short question.
0
Q: Continuity of a Free Functor

Daniél CárdenasIs Free Functor $Set ---> Ab$ continuous? Free Functor sends a set X into the free Abelian group FX generated by X.

Short title. Short question.
 
3:48 AM
0
Q: Math permutation and Combination

user290518How many 4-elements subset of a set containing numbers 1 to 9 have atleast two odd number as elements?

Short question.
 
0
Q: Guidelines for use of android-studio tag

Kevin KrumwiedeAre there any guidelines for the use of the android-studio tag? If so, where are they? The tag info currently contains a description of Android Studio itself, not the tag. I ask because Android newbies using Android Studio often tag their questions with android-studio even though their questio...

 
0
Q: What is a combined function and what factors affect its domain.

Aviry24Can someone please explain to me what a combined function is and how it is different from a composite function. From my understanding a combined function would be (f+g)(x) = f(x)+g(x) and a composite function is (f+g)(x)=f[g(x)] Also what factors affect a combined functions domain?

0
Q: green's theorem relating to calculating area

user3874530http://imgur.com/J8QUiAt In this problem, I have to find the area of that blob. pretty much I have to see if Nx-My is equal to 1. For the first choice, it IS equal to 1, yet the answer key says it is 4?

Short question.
Welcome to Math.SE, Aviry24. Tag (definition) should not be the only tag a question has. Please add a tag for a subject area to which the question belongs. (autocomment)Normal Human 20 secs ago
0
Q: Group Theory Colorings

AB818What are the colorings of a Cuboctahedron for up to $j$ colors? The colorings are the same if they can be rotated to be the same. I know I must use Burnsides Lemma, how? The Cuboctahedron is not fully symmetric

Short title.
Short question.
0
Q: Suppose exist a $[n , k , d ]$ liner block code like $C$ can you get hint an idea to prove exist a $ [ n , k , d-1 ] $ liner block code like $C' $?

AmirSuppose exist a $[n , k , d ]$ liner block code like $C$ can you get hint an idea to prove exist a $ [ n , k , d-1 ] $ liner block code like $C' $?

Short question.
0
Q: To check Convergence of series

Taylor TedHow do i check convergence of series whoose general term is given by $(\frac {n}{n+1})^{n}$. I have tried ratio and root tests but they don't furnish any results. Thanks

Short question.
0
Q: ARMA and causal process

DreamerHi can someone please explain to me what Causal process means and how to make sure a ARMA process is causal? For example, I have this equation: $X_t = 7X_{t-1} + aX_{t-2} + 3Z_t +9Z_{t-1}$ Thank you.

Consider adding a tag for a broader subject area to which the question belongs. Some of these tags might fit. (autocomment)Normal Human 21 secs ago
 
4:22 AM
0
Q: Should these two questions be considered duplicates?

kevinarpeIt seems there is an incredible amount of overlap between these two questions. Specifically, some of the answers in both may be useful for the other. Does this qualify as a duplicate? If so, is the policy to always select the newer question as a duplicate? Troubles with SureFire plugin: -"Th...

 
0
Q: Two sets S and T are disjoint and compact in a normed vector space. Define $f(S,T)=inf\{||s-t||:s \in S, t \in T\}$.

chloewennTwo sets S and T are disjoint and compact in a normed vector space. Define $f(S,T)=inf\{||s-t||:s \in S, t \in T\}$. Are there elements $s \in S$ and $t \in T$ s.t. $f(S,T)=||s-t||$?

Short question.
0
Q: Joint Probabolity

hizza waseemA fair coin is tossed three times. Let X be the number of heads among the first two tosses and Y be the number of heads among the last two tosses. What is the joint probability mass function of X and Y? What are the marginal probability mass function of X and Y? (i.e. p_x (x)and p_Y (y)?) Find...

Short title. Short question.
0
Q: How to express an inequality in form of LMI in Matlab, and use the feasp function?

vikiboyThis is my LMI constraint : (P1) + (beta^2) * R * R' > 0 where P1 = A' * (I + F*C)' * P + P*(I+F*C)*A + A'*C' * G'*(Py) + (Py) * G * C * A - C' * (Pk)' - (Pk) * C + 2I R = P + P * F * C + (Py) * G * C beta = some constant A' means the transpose of A matrix. A * B means matrix multiplicatio...

Welcome to Math.SE, vikiboy. This site uses MathJax formatting of formulas. More tips here. (autocomment)Normal Human 21 secs ago
0
Q: Number of Diagonalizable Matrices

AB818How many diagonalizable matrices are there under the field $p$, where $p$ is a prime?

Short question.
 
4:49 AM
0
Q: Expectation and the Survival Function: Measure Theory

AaronI have an example from class notes that I do not understand and would appreciate some clarification. Particularly, I haven't found a direct explanation online or in my text with regards to the limits of integration for the following example. If $X$ is a nonnegative random variable, then \begin{...

Question contains step by step.
 
5:01 AM
0
Q: Using variation of parameters method to solve ODE

CollapticI have the following ODE: $$y^{''} + 4y^{'} + 3y = 65cos(2x)$$ I first used the method of undetermined coefficients to find the solution to this equation, which was relatively easy, and found that the solution was. $$C_1e^{-3t}+C_2e^t+8sin(2x)-cos(2x)$$ where to the my $y_h =C_1e^{-3t}+C_2e^t$, I...

Questions tend to get more attention when they have a tag for a broad area of mathematics relevant to the question. Some of these tags might fit. (autocomment)Normal Human 21 secs ago
0
Q: Basic quadratic calculus

DreamerGiven $$ AX^2+2X-1=0 $$ What value of A would make the absolute value of both roots bigger than 1? I found the roots using quadratic formula but not sure what to do from there. Thanks!

Short title. Title contains basic. Short question.
0
Q: Is there any correlation between approximation trendline parameters?

LoomyBearLet's say I have two data sets (x,y) and (p,q) and two approximation trendlines: Logarithmic: y = b*ln(x) + a Linear: y = bx + a Let's say I applied logarithmic approximation to both data sets, so I have this: y = b1*ln(x) + a1 p = b2*ln(q) + a2 where: b1 > b2 After I've applied linear ap...

This site uses MathJax formatting of formulas. More tips here. (autocomment)Normal Human 21 secs ago
0
Q: $f''(x)+f(x)=0$

CrimeFind the general solution of $f''(x)+f(x)=0$ I know it has to be cosine, sine or the exponential, but I was wondering if there was a general form for the solution before applying the initial conditions. Thanks.

0
Q: Limits of integration in probability question

QualityI have a few questions in regard to the following example given and corresponding solution in Wackerlys Mathematical Statistics; Suppose $Y_{1}$ and $Y_{2}$ have jdf given by $f(y_{1},y_{2})=k(1-y_{2})$ if $0 \le y_{1} \le y_{2} \le 1.$ , $0$ elsewhere Then it asks to find (1) the value of k ...

Words such as question do not add information to titles. Please edit the title so that it better describes the specifics of your question. Do not hesitate to make it longer or include a formula if needed. More tips here. (autocomment)Normal Human 21 secs ago
0
Q: Barycenter of a measure

IdonknowSuppose that $X$ is a Banach space and $\mathcal{F}(X) = \overline{span}(\{ \delta_x : x \in X \})$, where $\delta_X : Lip(X) \rightarrow X$ and $Lip(X)$ is the set of real-valued Lipschitz functions on $X$. Define $\beta : \mathcal{F}(X) \rightarrow X$ given by $$\beta(\mu) = \int_X id_X du$$...

Short title.
0
Q: Approximating the integral of a binomial series

dirtcrazyI need help approximating the following integral to four decimal places using Taylor Series, I also need help approximating the error. Thanks. \int_{0}^{2}(1+2x)^{-1/5}

0
Q: Define f: A $X$ B as f(a,b) $\to$ a. Prove it is bijective.

user3552680I get that I have to show that it is injective and surjective. But I'm confused as to how to show that with an ordered pair to a single element.

Short question.
This site uses MathJax formatting of formulas. More tips here. (autocomment)Normal Human 21 secs ago
0
Q: Reduce a trigonometric expression 2.0

dblissBy what trigonometric trick does \begin{align} \sin\alpha\Bigg[\cos(\omega t + \varphi)+\frac{\cos\alpha\sin(\omega t + \varphi)}{\sin\alpha}-\bigg(\cos(\varphi)+\frac{\cos\alpha\sin(\varphi)}{\sin\alpha}\bigg)e^{-\omega t/x}\Bigg]\\ \end{align} reduce to \begin{align} \bigg[\sin(\alpha+\omega...

Title contains 0.
0
Q: How to attack this problem with a volume that is hard to interpret / sketch,

user290432Part 1 Show that the equation for the tangent plane to the ellipse $x^2/a^2+y^2/b^2+z^2/c^2=1$ at the point $(x_0,y_0,z_0)$ is given by $xx_0/a^2+yy_0b^2+zz_0c^2=1$ (*). Part 2 Let $V(x_0,y_0,z_0)$ denote the volume bounded by the planes $x=0,y=0,z=0$ and the plane (*) for $x_0>0,y_0>0,z_0>0$....

Title contains problem, hard.
0
Q: PA2 (Peano Arithmetic in 2º order logic and categoricty)

Raúl Aparicio BustilloSecond order logic implies categoricity in peano arithmetic. But why are the models isomorphic to the standard model of aritmhetic and not to another non standard model, for example?

0
Q: How would I go about solving the following simultaneous congruence:

dd123How do I solve the following simultaneous congruence: 25x congruent 18 (mod 48) x congruent 11 (mod 35). *I don't know how to make the congruence symbol on my keyboard

Short question.
0
Q: Beta Distributions: Probability

JohnMy teacher did this problem in class however he did not go over the steps. He gave the answer of .1808. Can someone guide me through the steps? I am really struggling with Beta Distribution. Suppose the proportion of new new businesses in the city of Barryville that fail within one year is a be...

Short title.
Consider adding a tag for a broader subject area to which the question belongs. Some of these tags might fit. (autocomment)Normal Human 24 secs ago
0
Q: Function approaches zero but derivative doesn't

Shah Rukh QasimIf: y=f(x) and y=0 when x-> ∞ Is it possible that: d/dx(y) is not equal to zero when x-> ∞ And prove it!

Short question.
0
Q: High school Math: confusion about the basic probability

kimberlyI am confuse in the following two scenerio In a bag of 3 apples and 3 oranges. You pick 2 items from the bag. 1) what is the probability that you will have 2 apples? 2) what is the probability that you will have 1 apple and 1 orange? My attempt: 1) P(2 apples)=P(1st apple)xP(2nd apple)=3...

Welcome to Math.SE, kimberly. This site uses MathJax formatting of formulas. More tips here. (autocomment)Normal Human 21 secs ago
0
Q: Average of decimal digits

Michael LeeI define the function $d_{\mathrm{avg}} : [0, 1]\to [0, 1]$ such that for $0.x_1x_2x_3\cdots$ the decimal expansion of $x$ (defined such that $\nexists N : x_k = 9$ for all $k \geq N$), $$d_{\mathrm{avg}} : x\mapsto \lim_{n\to \infty} \frac{1}{n}\sum_{i=1}^{\infty} x_i$$ How would I show that the...

Short title.
0
Q: Skew-symmetric matrix

Archis WelankarI have to prove that determinant of skew- symmetric matrix of odd order is zero and also that its adjoint doesnt exist. I am sorry if the question is duplicate or already exists.I am not getting any start.I study in Class 11 so plase give tge proof accordingly. Thanks!

Short title.
0
Q: Proving Convexity for a function f(x) = 1 / g(x)?

MikaelSo I have a function g that maps from some subspace, S, of R^n to R. g is concave such that g(x) > 0 for all x in this subspace, S, of R^n. f(x) is defined as f(x) = 1 / g(x) and the question is to show that f(x) is convex. How would I do this by the definition of convexity? My try: As g is ...

This site uses MathJax formatting of formulas. More tips here. (autocomment)Normal Human 21 secs ago
0
Q: Find a matrix X∈V such that U∩W=span{X}.

Ernestomath problem on here Have been trying this problem for 4 hours still can't figure it out.

Short question.
 
5:52 AM
posted on November 16, 2015 by Scott Morrison

Long time MathOverflow users may be aware of a user who has been permanently banned from both MathOverflow and the entire StackExchange network. Despite the ban, they have been rather persistent and ...

 
@AndrewT. Any idea why the feed sometimes formats posts as if there were blog posts?
Such as the one above.
0
Q: Questions about ZFCU (ZFC with atoms) in second order logic

Raúl Aparicio BustilloIs categorical ZFC2U?? Is expressable the cardinality of a set A from ZFC2U from a sentence in second order logic?

Short question.
@StackExchange This is the "normal" post by a feed, for comparison.
Hm, and now the post by Morrison is deleted.
0
Q: Think I invented a formula can u guys verify ("a" square divided by 8)

VishnuLet the sum of 3 sides of a reactant le be "a" find the max area possible with these dimensions My identity I derived is "a"square divided by 8 2x+y=a Area =x(a-2x) X(a-2x)-area= 0 Xa-2x2-area=0 Put in derivatives cause area is the max when slope is 0 A-4x=0 X=a/4 Y= a-2(a/4) Y is a/2 A/4*a/...

Welcome to Math.SE, Vishnu. This site uses MathJax formatting of formulas. More tips here. (autocomment)Normal Human 20 secs ago
 
@NormalHuman Sorry, no idea at all (that's the first time I see)
 
I see a pattern: these are meta sites of non-stackexchange domains. Hm.
0
Q: rings problem-- Consider the equation x^2+y^2=n

Fengdi ZhaoConsider the equation x^2+y^2=n 1. find all solution to x^2+y^2=2 2. suppose that n is a positive integer congruent to 3 modulo 4. Prove that the equation x^2 + y^2 = n has no such integer solutions.

Welcome to Math.SE, Fengdi Zhao. This site uses MathJax formatting of formulas. More tips here. (autocomment)Normal Human 20 secs ago
0
Q: What we can say about the convergence of following double series

chetan joshiWhether the following double series is convergent or divergent?

Short question.
0
Q: Looking for your favorate proof that $\prod_{n\in P} n=4/pi^2$

0.5772156649...What is your favorite proof that $$\prod_{n\in P} n=4\pi^2\text{?}$$

Short question.
 
That's why I just tested posting meta.MO, but not repro'd. A bug on feed itself maybe?
 
0
Q: find a matrix whose row echelon form is the same as the row echelon form of its transpose

MariavCan someone help me with this textbook question: can you devise a nonzero matrix whose row echelon form is the same as the row echelon form of its transpose? Thank you.

Short question.
0
Q: the number of objects from a collection of N objects that possess exactly m of the properties

Liu_CSLet $N_m$ denote the number of objects from a collection of $N$ objects that possess exactly m of the properties $a_1,a_2,\ldots,a_r$. Generalize the principle of inclusion-exclusion by computing $N_m$ as the following form and please explicitly give the $s_k$. $N_m= \sum\limits_{k=m}^r(-1)^{k-m...

Question contains please.
Weird, I can't even onebox now.
0
Q: How can I rearrange ($a$ x $b$)^$c$=$d$ to make $c$ the subject

Bailey FYIs it at all possible to rearrange this equation to make c the subject. My dad and I believe it is possible but I am unable to correctly rearrange for c.

Short question.
 
6:16 AM
(forgot that the post has been already deleted)
(can't onebox that Russian meta)
-2
Q: Формулируем «100 и более баллов» :: Часть 2

QwertiyЧасть 1.Хотим ли мы видеть любое упоминание "и более" в тексте?Нет А теперь продолжение: Часть 2Хотим ли мы акцентировать внимание на слове "Ответ", поставив его на первое место? Первое слово в описании знака акцентирует внимание на неком предмете или действии. Вероятно, в случае достижения от...

(had a difficulty oneboxing that post D:)
 
0
Q: Are the subgroups of Sn symmetric?

BananaI have a quick question. Are all the subgroups of $S_n$ symmetric as well?

Short question.
Maybe this chatroom is hitting some oneboxing limit...
0
Q: Interpolating $\frac{d^kf}{d(g(q))^k}=k!h(x)$

Ben S.Suppose I have a function $f(z)$ defined for every complex $z$ with real part strictly greater than $0$, and that I have been able to show $\frac{d^kf}{d(g(q))^k}(k)=k!h(q)$ for every positive integer $k$ and expressions $g$ and $h$ ($q$ is a fixed parameter). Are there any general results or met...

Short title.
So, not clear what causes this, at all. Oh well.
0
Q: Cohomology ring is commutative in the graded sense.

user233017Let $X$ be a CW complex. How do I see that the cohomology ring $H^*(X)$ is commutative in the graded sense:$$x \cup y = (-1)^{pq} y \cup x \text{ if }\deg x = p \text{ and }\deg y = q?$$

Short question.
0
Q: How to change the equation to polar form?

LearningMathCompute $\displaystyle\int^\infty_{-\infty} dx\displaystyle\int^\infty_{-\infty} dy\displaystyle\int^\infty_{-\infty} dz \delta(\sqrt{x^2 +y^2+z^2} - R)$.

Short question.
0
Q: Branches of the logarithm function problem

Razor1692Let $D$ be a domain in $ C$\ {0} such that the annulus {z$\in$$ C$ such that 1<|z|<2} is contained in $D$.Prove that there is no branch of the logarithm defined in $D$. My Attempt: Select a point Z'$\in$ {z$\in$$ C$ such that 1<|z|<2} such that Z'=-x where 1 < x< 2. Then Log(Z')=lnx +i$ \pi$ . ...

Title contains problem. Question contains please, please.
0
Q: Which branch of logarithm is the following logarithm defined on

IllustionistI need the find at which branch of the logarithm the $log(z - z_0)$ defined on, where $z_0 \in \mathbb{C}$. I need it for a problem that I am solving.

Short question.
0
Q: What is the formula for ARMA?

DreamerI wonder what the formulas are for ARMA(p,q), ARMA(p,0), ARMA(0,q)? I was asked to calibrate for a ARMA(3,0). I found 3 parameters using matlab but I am not sure how to formulate the equation...

Questions tend to get more attention when they have a tag for a broad area of mathematics relevant to the question. Some of these tags might fit. (autocomment)Normal Human 21 secs ago
0
Q: Find all real polynomials $P(x)$ which satisfy the equation$ P(x)P(-x)=P(x^2-1)$

Heinz KloubeFind all real polynomials $P(x)$ having only real zeros and which satisfy the equation $$P(x)P(-x)=P(x^2-1)$$ Please explain me the process and refer some books to learn polynomials. Thanks in advance !

Question contains please.
0
Q: Question on Group-Homorphism

learnmoreLet $G=\{$ \begin{bmatrix} a & b \\0 & a^{-1}\end{bmatrix} :$a,b\in \mathbb R; a>0\}$ $N=\{$ \begin{bmatrix} 1 & b \\ 0 & 1 \end{bmatrix} $:b\in \mathbb R\}$ Prove that $G/N$ is isomorphic to $(\mathbb R,+)$ and $G/N$ is isomorphic to $(\mathbb R^{+},*)$. I have to get a onto homomorphism fro...

Words such as question do not add information to titles. Please edit the title so that it better describes the specifics of your question. Do not hesitate to make it longer or include a formula if needed. More tips here. (autocomment)Normal Human 21 secs ago
0
Q: Collection of Subsets

GGmathThere are integers $m, n > 0$, and a collection $S$ of distinct subsets of some ambient set $A$, each of size at most $m$. Assume $|S| > (n − 1)mm!$. Prove that there exist $n$ sets $A_1, . . . , A_n ∈ S$ such that the intersections $A_i ∩ A_j$ are the same for all pairs $(i, j)$ where $i$ is not...

Short title.
 
6:55 AM
6
Q: Reword close message -- better reflect policy on sister sites

tripleeeI would like to propose changes to the close messages for off-topic questions. The close message when a question is put on hold as off-topic contains the following guidance for questions which were not selected for migration. Questions about general computing hardware and software are off...

0
Q: a case of vote rigging?

cnstHow is it possible that this question, where it's not even clear what the guy is asking or attempting to do, and ignoring a single request for clarification, has generated a +8 in a mere 39 views (all of these +8 being in a single 24h period), yet the very single answer that was there all along h...

 
0
Q: Given A is a linear transformation find a non zero polynomial such that p(A)=0

RagingBull (a) The space of all linear trnasformations from V->V has dimension n^2. Consider I,A,A^2,A^3,.....A^(n^2). Then this collection forms a linearly dependent set.So we have a non trivial relation amongst these which will give us a polynomial such that p(A)=0. (b) Don't know how to proceed.

This site uses MathJax formatting of formulas. More tips here. (autocomment)Normal Human 25 secs ago
 
7:12 AM
0
Q: How to calculate Nth permutation of a set when sorted by sum?

Tomasz TkackiI have a given set (0<size<10^6) of numbers (up to 10^9). How do i calculate Nth (0<N<10^6) subset when sorted by sum

Short question.
0
Q: Justification for arc length formula using Riemann Sum

HmmmmI recently found myself rather delighted after I had sort of proved the arc length formula for a curve $x \mapsto f(x)$, $x \in [a;b]$. However, I realized that I didn't really know how to argue rigorously why the limit yields the definite integral (I just need a good argument). What I have on my...

Questions tend to get more attention when they have a tag for a broad area of mathematics relevant to the question. Some of these tags might fit. (autocomment)Normal Human 21 secs ago
0
Q: complete statistics

DominikaLet X be the sample from the uniform distribution $U(0,\theta)$ where $\theta \in (0,\infty)$ Show that the statistic $T(\textbf{X})=X_{n:n}$ is complete. Thanks a lot for all your help.

Short title.
0
Q: Some boys in the class are taller than all the girls

Mithlesh UpadhyayIdentify the correct translation into logical notation of the following assertion. Some boys in the class are taller than all the girls Note: taller $(x, y)$ is true if $x$ is taller than $y$. $(∃x)($boy$(x)→(∀y)($girl$(y)∧$taller$(x,y)))$ $(∃x)($boy$(x)∧(∀y)($girl$(y)∧$taller$(x,y)))$ $(∃x)...

Question contains please.
Tagged proof-explanation.
0
Q: What is the correct way to formulate dual function from the Lagrangian?

Fence JumperSuppose I am given the problem $$\min c^Tx\\ s.t. Gx \leq b\\ Ax = b$$ The Lagrangian is: $$L(x,\lambda,\nu) = c^Tx + \lambda^T(Gx -b) + \nu^T(Ax-b) = (c^T + \lambda^TG + \nu^TA)x-\lambda^Tb -\nu^Tb$$ What is the correct way of thinking to formulate the dual function at this point? Two alte...

0
Q: I'm confused on how to do (x^2 + 1)/(x + 1) is O(x)?

CastFor me I (x^2+ 1)/(x+1) by x^2 since that's the term with the highest exponent making it: (1+(1/x^2))/((1/x)+(1/x^2)) but I'm not sure where to go from there. I looked at textbook solution and they have: (X^2+1)/(x+1) = (x-1+2)/(x+1) but I'm not sure how they came up with that.

This site uses MathJax formatting of formulas. More tips here. (autocomment)Normal Human 22 secs ago
0
Q: How to find the last one digit of 8^97?

dd123What will be the units digit of: 8^97 I know I will have to start like: 8^1, then 8^2, 8^3 etc... is there a shorter method to finding the units/digit?

Short question.
0
Q: A question to do with floor and tiles

JEFFREYBasically the question goes something like this: A floor of length 16m and width 4m is covered with tiles. The tiles are square tiles. You are required to find the area of the tiles and the number of tiles used.Sometimes they might ask you to find the greatest area of the tiles. So far this is ho...

Welcome to Math.SE, JEFFREY. Words such as question do not add information to titles. Please edit the title so that it better describes the specifics of your question. Do not hesitate to make it longer or include a formula if needed. This site uses MathJax formatting of formulas. More tips here. (autocomment)Normal Human 21 secs ago
 
7:42 AM
1
Q: Add "This is spam" as option for recommending deletion in the LQP Queue

MagischMore then once now it happened to me that I found a post that is clearly spam while reviewing Answers in the LQP Queue and subsequently had to go to the question, seek out the Answer (often among many) and manually spam flag it. Currently there is no way to flag something for spam directly from t...

 
0
Q: Help me solve the equation

he hehiI need solve the follow equation: a) A man to be building a table of 6 hours , while another man to do the same table needs 12 hours. For how many hours the two will perform the same work. b)A sportsman distance between countries A and B passed for a limited time. If moving with speed 35km / h w...

Words such as help do not add information to titles. Please edit the title so that it better describes the specifics of your question. Do not hesitate to make it longer or include a formula if needed. More tips here. (autocomment)Normal Human 22 secs ago
0
Q: Expected Value Diverges?

Tin PhanGiven the probability density function $f(x) = \frac{1}{x^2}$ for $x > 1$. The expected value of this function is: $$E[X] = \int_1^\infty \frac{1}{x} dx = \infty$$ Can someone please explain this? Is $E[X]$ only finite under certain condition? Is $\infty$ really the expected value for this probab...

Short title. Question contains please.
 
0
Q: When did "Name this thing" become a meme?

Robert HarveyI've been racking my brain trying to figure this out. What happened that caused "What is the name of this thing so I can go research it myself" become a meme? I've been a participant on Stack Overflow for quite some time now. I've seen a number of memes come and go, including The Great Communit...

-1
Q: 'Go to definition' doesn't work in anaconda

joeI installed anaconda in Ubuntu14.04, then I installed the keras using git clone https://github.com/fchollet/keras.git cd keras sudo python setup.py install but when I try to using the 'Go to definition' item in the right click menu to see the definition of that python module, th...

 
8:04 AM
0
Q: Branch cut problem, square root

user290433I am looking at $f(z)=\sqrt{1-z^2}$ and a branch cut on the real axis from $z=-1$ to $z=1$. Is it correct to say that $f(x+i\epsilon) = -f(x-i\epsilon)$ when $x\in(-1,1)$, $\epsilon\in\mathbf{R}$ and $\epsilon\to 0$? If this is true, is it also true that $f(x+iy)=-f(x-iy)$ when $y>0$? (The prob...

Title contains problem.
0
Q: How to bound the input parameters to a chaotic function to obtain exact result in a finite precision setting?

Ali ShakibaWhile I was reading the paper entitled (http://dx.doi.org/10.1109/ISCAS.2003.1204947) Kocarev, Ljupco, and Zarko Tasev. "Public-key encryption based on Chebyshev maps." Circuits and Systems, 2003. ISCAS'03. Proceedings of the 2003 International Symposium on. 2003. I encountered a table,...

Consider adding a tag for a broader subject area to which the question belongs. Some of these tags might fit. (autocomment)Normal Human 23 secs ago
0
Q: Littel bit confused dxdt or dtdx?

hichamgaussin a lemma of sobolev spaces, i found this integral: for $x\in \mathbb{R}^k$ and $t\in\mathbb{R} $ this intgral (verify somme properties) $$\int_{\mathbb{R}^k}\int_\mathbb{R}|{\frac{u(x+te_i)-u(x)}{t}}|^pdxdt$$ is there any problem here of $dxdt$ i mean the first integral is normally mast be i...

Title contains confus.
0
Q: How to set up linear programming problem for maximizing score of various combinations?

beginnerI have a sample data set that looks like this: x y w 1 1 5 1 2 1 6 2 3 1 7 3 4 2 8 4 5 2 7 5 6 3 5 6 7 4 6 7 8 4 5 8 x and y represent indices from datax and datay. w represents a score from comparing datax[x] with datay[y]. I want to maximize the total score (or w) from d, where each value ...

Title contains problem.
0
Q: Levi civita notation

DariusI'm having troubles with the Levi-Civita symbol. I understand what the normal epsilon-tensor means and how it works. But how do I interpret this: $\mathrm{{\epsilon_{i}}^{jk}}$ or $\mathrm{{\epsilon_{ik}}^{k}}$ ? Thank you!

Short title.
0
Q: The problem related to Exponential distribution.

Danny_KimQuestion: There are two batteries. A Battery's life is following the Exp(1/20) distribution. The other one's life is following the Exp(1/40) distribution. One day, a person randomly chose one battery. When the battery was used for 20 hours, find the probability that the battery can be alive 10 ...

Title contains problem.
0
Q: prove why RSA requires the use of distinct primes

dd123I need help proving why RSA requires the use of distinct primes. What I have done so far: I figured since I need to prove why RSA requires the use if distinct primes I am going to disprove the following: if p and q are the same number, n = pq c and are positive integers such that : cd is con...

This site uses MathJax formatting of formulas. More tips here. (autocomment)Normal Human 25 secs ago
0
Q: Given that xn includes interval [a,b] and limit of xn = z. Prove z includes interval [a,b].

SallyGiven that xn includes interval [a,b] and limit of xn = z. Prove z includes interval [a,b]. a<=xn<=b is given and I want to show a<=z<=b. Since limit of xn converges to z, abs(xn-z)<ε and z-ε

This site uses MathJax formatting of formulas. More tips here. (autocomment)Normal Human 22 secs ago
0
Q: SR Unit Conversion Speed

John SnoeWhat is 72,000 km/hour in Système Relativistique (SR) units? I am getting 1/14490 or 1/14490th of the speed of light.

Questions tend to get more attention when they have a tag for a broad area of mathematics relevant to the question. Some of these tags might fit. (autocomment)Normal Human 22 secs ago
0
Q: Soluion of LTV Equations

user290575Can anyone help me in finding solution of the LTV equation given by t\ddot{x}-\dot{x}+tx=0 or equivalently the LTV system \dot{x}_1 =-tx_2, \dot{x}_2 =-(1/)tx_1. Thanks for your time and help! Shah

Welcome to Math.SE, user290575. This site uses MathJax formatting of formulas. More tips here. (autocomment)Normal Human 20 secs ago
0
Q: Mathematical proof of an equivalence relation partitioning space?

Anjani GuptaI understand it intuitively but please help me with its mathematical proof. Thanks

Short question. Question contains please.
0
Q: Lifting of triangulation

Harry BarberIn "Complex Analysis 2: Riemann Surfaces, Several Complex Variables, Abelian Functions, Higher Modular Functions" and many other books is described a lifting of triangulations for branched covers between surfaces. Lifting process May we generalise this branched covers (open, discrete maps) bet...

Short title.
0
Q: Need help with simple math about currency

DrayneHi i got a quick question because im trying to figure out exactly how much money i will get when i take out money from the ATM in Prague (visiting from Sweden). The last time i took out money it was 4000 Czech Koruna and the amount it took from my swedish bank is 1.529,36. The currency exchange...

Welcome to Math.SE, Drayne. Words such as help do not add information to titles. Please edit the title so that it better describes the specifics of your question. Do not hesitate to make it longer or include a formula if needed. More tips here. (autocomment)Normal Human 20 secs ago
 
9:01 AM
-1
Q: Matlab: how to create a semantic tree/graph?

PamI want to create a semantic tree: can you give me some suggests? thanks

 
0
Q: How to Solve a differential equation with both x and y?

John SmithSolve $\dfrac{dy}{dx}=\dfrac{y-3}{y^2+x^2}$ given that it passes through $(0,1)$. Right now I do not yet know how to solve differential equations with both $x$ and $y$ that you cannot separate. Please help. Thanks.

Question contains please.
 
0
Q: Notify others users to join the chat room for discussion

JohnWhy doesn't Stack Overflow provide a feature which can invite other users to a chat room? I know that if there are too many comments but the problem still not yet solved, we can move the discussion to chat in order to make the page clean. However, some of the discussion might not come out with ...

 
0
Q: Problem with change of variables (with regard to kernel density estimation)

xxxI'm trying to understand why \begin{equation*} \begin{split} E(R(\hat{f}'')) & = \frac{1}{nh^6} \int \int K'' \left( \frac{x-y}{h} \right)^2 f(y) dy dx \\ & \quad + \frac{n(n-1)}{n^2 h^6} \int \int \int K'' \left( \frac{x-y}{h} \right) K'' \left( \frac{x-z}{h} \right) f(z) f(y) dz dy dx \\ & =...

Title contains problem.
0
Q: prove the following by induction

dd123Suppose M = p1*p2*...pn, where p1,...,pn are distinct primes Prove by induction that, for all positive integers n, if gcd( a, M) = 1, then a^(p1-1)(p2-1)(pn-1) is congruent to 1 (mod M): I have no clue where to even start I.E base case. please help!

Question contains please.
 
9:30 AM
0
Q: Finding the percent of a division fast and mentally

Math is fun3/8= (0.125*3) = 0.375 = 37.5% is easy to calculate mentally but is there a better way to find the percent of these divisions fast and mentally? 3.5/8 4.5/7

Short question.
0
Q: >Q What is the remainder when 1!+2!+3!+4!+5!+.......+50! is divided by 5!?

justin takro Q What is the remainder when 1!+2!+3!+4!+5!+.......+50! is divided by 5! MyApproach $1$+$2$+$6$+$24$+$5$!/$5$!+$6 . 5$!/$5$!+$7$ .$6$ . $5$!/$5$!....so on $33$+$1$+$6$+$42$+...... I am not getting correct Ans as the solution is getting complex. Can anyone guide me how to approach the ...

Title contains !?.
0
Q: Exercise 3.6 from rudin's real and complex analysis

pxchg1200Let $m$ be Lebesgue measure on $[0,1]$, and define $\|f\|_{p}$ with respect to m.Find all functions $\Phi$ on $[0,\infty)$ such that the relation $$ \Phi(\lim_{p\to 0^{+}}\|f\|_{p})=\int_{0}^{1}\Phi\circ f dm $$ holds for every bounded,measurable,positive $f$.Show first that $$ c\Phi(x)+(1-c)\Phi...

Title contains exercise.
0
Q: Suppose f is defined on an open interval containing x and f'' exists. Show that $lim_{h\to 0}\frac{f(x+h)+f(x-h)-2f(x)}{h^2}$.

Kevin McDonoughShow that P is the taylor polynomial for f at c, for example: $$P(x)=\sum_{k=0}^n \frac{f^{(k)}(c)}{k!}(x-c)^k$$ Sorry the whole problem did not fit in the title. Other Important information: f is an element of $C^n(I)$ where I is an interval, and c is an element of I. My Work: I plan to comp...

Question contains please.
-1
Q: What is infinity?

ShalemXI have a question about infinity. There was a thought coming to my mind after watching a video about infinity. Could it be that infinity is somekind of movement? Now let me explain this thought. There is this paradox with the Grand Hotel. It is full. But due to its infinity someone can come and c...

Short title.
 
0
Q: Evaluating limits using natural logarithm

DanaSFind $$ \lim_{x\to0}\left(\frac{1+x2^x}{1+x3^x}\right)^\frac1{x^2} $$ I have used natural logarithm to get $$ \exp\lim_{x\to0}\frac1{x^2}\ln\left(\frac{1+x2^x}{1+x3^x}\right) $$ After this, I have tried l'opital's rule but I was unable to get it to a simplified form. How should I proceed fro...

0
Q: Formatting in excerpt. Strip all of it or make it work

HarisFew days back I was editing tag wikis when I realized that formatting doesn't work in the excerpt section. I came across some tag wikis in which the formatting was showing (like asterisk and all to make it bold or italic) and it was looking dirty. I had asked a question as to why formatting does...

 
9:51 AM
0
Q: Combinatorial interpretation of $ \begin{Bmatrix} n\\ k \end{Bmatrix} =\frac{1}{k!}\sum_{j=0}^{k}(-1)^{k-j}\binom{k}{j}j^n$

portalI see that the below formula is the explicit formula of the Stirling numbers of the second kind. I know that the Stirling number of the second kind is the number of ways to partition set of $n$ objects into $k$ non-empty subsets. But, I don't at all see from where the below formula comes from. Cl...

Title contains \begin.
0
Q: Rolling and rerolling dices

Albert MasclansI roll 10 initial dices. Whenever I roll a $6$ I roll 5 more dices. What's the expected number of dices I'm going to throw? I was thinking about it when trying to create a die system, but can't solve the problem.

Short title.
0
Q: a square inscribed right triangle

user3656801I recently came across this problem. the problem The hypothenuse of the triangle has a length of 4, is base (highlighted in green) a length of x and the last side (highlighted in red) a length of y I found this two equations to solve the problem. First of all, x²+y²=16 (considering Pythagore'...

This site uses MathJax formatting of formulas. More tips here. (autocomment)Normal Human 26 secs ago
0
Q: Weighted L1 norm

sheepI try minimizing the following expression : $ V(x)=\sum_{i=1}^n|x_i - u|w_i $ I need to find u that minimize this expression. I know how to do it for w=1 (which is is the median). Any ideas for the general case ?

Short title.
 
10:10 AM
0
Q: Trigonometry equation, odd-function.

PerSo I have the following equation: $\sin(x-\frac{\pi}{6}) + \cos(x+\frac{\pi}{4})=0$ It should be solved using the fact that Sin is an odd function, I can not really get the gripp of how and what I need to do? Any sugestions?

Short title.
 
0
Q: How to migrate a question after it was put on hold

docendo discimusThe following question was put on hold as off-topic before I first saw it. So when I saw it, I thought it should be migrated to Cross Validated but since it was put on hold already, I couldn't vote to move it to the proper (imo) site. I decided to flag it for moderator attention commenting that i...

 
0
Q: Prove or Disprove: if $\lim\limits_{n \to \infty} (a_{2n}-a_n)=0,$ then $\lim\limits_{n \to \infty} a_n=0.$

F1sargyanProve or Disprove: if $\lim\limits_{n \to \infty} (a_{2n}-a_n)=0,$ then $\lim\limits_{n \to \infty} a_n=0.$ I don't think that this is true. and I'm trying to think about counterexample, but couldn't figure out a mathematical form of the sequence that I thought about, its a sequence where certai...

Title contains \limits, \limits.
0
Q: Interesting facts/ proofs about rational and irrational numbers

Tk706We got set some work to find some interesting facts or proofs regarding rational and irrational numbers. I wonder if anyone could offer some insight or recommend a good book/ website to look at.

Title contains interesting. Tagged proof-explanation.
0
Q: Summation Proof (Combinatorics)

Kelsey ArthurThe question says to prove that $$\sum_{n_1+n_2+n_3=n} \binom{n}{n_1*n_2*n_3}*(-1)^{n_1-n_2+n_3} = 1$$ where the summation extends over all nonnegative integral solutions of $$n_1+n_2+n_3=n.$$ I used the multinomial theorem to get: $$(x_1+x_2+x_3)^n = \sum_{n_1+n_2+n_3=n} \binom{n}{n_1*n_2*n_3...

Short title. Tagged proof-verification.
0
Q: Two expressions using three-digit floating point arithmetic with rounding?

Mithlesh UpadhyayWhat is the result of evaluating the following two expressions using three-digit floating point arithmetic with rounding? $(113. + -111.) + 7.51$ $113. + (-111. + 7.51)$ $9.51$ and $10.0$ respectively $10.0$ and $9.51$ respectively $9.51$ and $9.51$ respectively $10.0$ and $10.0$ respectively...

Question contains please.
0
Q: How can I estimate the volume of the region inside an open bounded set in dimension n at distance less than 1/n from its boundary?

mraHow can I estimate the volume of the region inside an open bounded set in dimension $n$ at distance less than $1/n$ from its boundary?

Short question.
0
Q: Find $\lim\limits_{n\to\infty}\frac{x_{n+1}}{x_n}! $ where $x_n=x_{n-1}+x_{n-2},x_1=1,x_2=2$

display_errorFind $\lim\limits_{n\to\infty}\frac{x_{n+1}}{x_n}! $ where $x_n=x_{n-1}+x_{n-2} ,(n>2),x_1=1,x_2=2$ $x_n=x_{n-1}+x_{n-2}$ $x_{n+1}=x_{n}+x_{n-1}$ From the first recurrence relation, $$x_n=\frac{3+\sqrt{5}}{5+\sqrt{5}}\left(\frac{1+\sqrt{5}}{2}\right)^n+\frac{3-\sqrt{5}}{5-\sqrt{5}}\left(\frac{...

Title contains \limits. Tagged proof-verification.
0
Q: Adjusted exponential reduction

HoleneI have a problem where an amount, lets say $x$, are reduced by some percentage $p$ over a duration of $n$ descrete iterations. I want to calculate a factor $f$ such that $x_i = f_i \cdot x$, and the $n$th factor reduces $x$ to $x_n$ by $p$ percent: $f_n = (1-p)$. This gives a sequence of factors ...

Short title.
0
Q: How to calculate these limits?

Andi ZhangHow to calculate $\lim _{n\rightarrow \infty }\dfrac {n!}{n^{n}}=0$, $\lim _{n\rightarrow \infty }\sqrt [n] {n}$, and $\lim _{n\rightarrow \infty }\sqrt [n] {\dfrac {1}{n}}$?

Short question.
1
Q: Prove or disprove: If $\sum{a_n}$ with $a_n>0$ is convergent then $\sum{a_n^2}$ is always convergent.

user290591So that's the question. But I am unable to find a counter example if it's false. Or how should I proceed to prove if it is true? I am confused please help. Many thanks.

Short question. Question contains please.
0
Q: in $N$ to $\delta_{0}$f

zahrashow that $x$ is a power of 2 is equivalent in $N$ to $\delta_{0}$formula.(use the facts that $y $divides $x$ and 2 divides$y$are are equivalent in $N$ to $\delta_{0}$formulas.

Short title. Short question.
0
Q: Covariance same implies linear transformation

user23510Suppose, for two positive random variable $X$ and $Y$, and a convex function $g$, $$Cov(X,Y)=Cov(X,g(Y)).$$ Then how to show that $g(Y)$ is a linear convex function of $Y$?

Short question.
0
Q: SELLING PRICE QUESTION

rstProblem A man bought three tables or rs. $2500$. He sell the first at $5 $ % loss, second at $5 $ % profitable third at $10$% profit. Find the cost price of each table if on the whole he neither gains nor losses. Solution Let cost price of first table be x Let cost price of second table be y...

Words such as question do not add information to titles. Please edit the title so that it better describes the specifics of your question. Do not hesitate to make it longer or include a formula if needed. More tips here. (autocomment)Normal Human 23 secs ago
 
10:57 AM
1
Q: $\color{red}{\text{Math Processing Errors}}$ in the Close as Duplicate Preview

draks ...I see plenty of $\color{red}{\text{Math Processing Errors}}$ in the close as duplicate preview:

 
0
Q: If f(Cl(A)) is contained in Cl(f(A)) then f is a hoemomorphism

SamHow to prove this statement where f is a mapping between topologial spaces X and Y and f(Cl(A)) is contained in Cl(f(A))

Short question.
0
Q: Why is tree not uniquely possible with given preorder and postorder traversal?

Mithlesh UpadhyayConsider the label sequences obtained by the following pairs of traversals on a labeled binary tree. Which of these pairs identify a tree uniquely? preorder and postorder inorder and postorder preorder and inorder level order and postorder I've read that inorder is necessary to draw unique ...

Question contains please.
0
Q: How to prove $\lim\limits_{x\rightarrow\infty}f(x)=0$ and $\lim\limits_{x\rightarrow\infty}f''(x)=0\Rightarrow\lim\limits_{x\rightarrow\infty}f'(x)$?

João Victor Bateli RomãoFrom a exercise list: Let $f:[a,+\infty)\rightarrow \mathbb{R}$ with continuous second derivative and such that $\lim\limits_{x\rightarrow\infty}f(x)=0$ and $\lim\limits_{x\rightarrow\infty}f''(x)=0$. Prove that $\lim\limits_{x\rightarrow\infty}f'(x)=0$. Does anybody has a hint or a solution? I...

Title contains \limits, \limits, \limits.
0
Q: facts about the the number 1

Tk706We have been set some work to find out some detail proofs or facts about the number one. Can anyone help or tell me where to look? This is at university level.

Short title. Title contains 1. Short question.
0
Q: Show that open set can be represented by countable union of cubes?

lanse2pty$\Omega\subset R^n$ is a open set, how to show $\Omega=\bigcup\limits_{n=1}^\infty T_n$,$T_n$ is n-cube.

Short question.
 
11:13 AM
0
Q: Isn't this question way too broad?

OldskoolI came upon the following review audit today: http://stackoverflow.com/review/close/10218382 After analyzing the question I opted to vote to close the question for being too broad. As it stands, I find the question very vague and lacking details. The problem described "I'm getting an URL not fou...

 
0
Q: Rewriting this expression in the form of $n^2$.

BlueSkyThis expression is written under a radical and I need to take it out. So it needs to be in the form of $n^2$ to come out. ($\sqrt{n^2}=|n|$) $$(a+b+c)(a+b-c)(a-b+c)(-a+b+c)$$

Short question.
0
Q: question on hormonic power

Sarnavo SarkarProve that $1/2^2$+$1/3^2$.......+$1/2015^2$ is ~ 1

0
Q: Maximization issues

Prince MorI need help solving this maximization assignment. I am to find maximum $\rho_{k}$. All other variables are constants. \begin{equation} \begin{aligned} & \underset{ \rho_{1} \ldots \rho_{k}}{\text{maximise}} \frac{P_T\sum^{R}_{k = 1}\left[ \frac{|h_{k}g_{k}|\rho_{k}^2 (1-\rho_{k})(\beta\rho_{k} P...

 
-1
Q: magento.dingding.in?

alexandreI just found by googling the page http://magento.dingding.in/ I doubt that this is official and just wanted to share my discovery since it's maybe dangerous for others to use that page?

 
Short title.
Words such as question do not add information to titles. Please edit the title so that it better describes the specifics of your question. Do not hesitate to make it longer or include a formula if needed. More tips here. (autocomment)Normal Human 51 secs ago
0
Q: integrating continued fractions

Jon Mark PerryHow do you integrate: $$\int \underset{j=1}{\overset{\infty}{\LARGE\mathrm K}}\frac{a_jx^j}{b_j}dx=\int x\cfrac{a_1}{b_1+\cfrac{a_2x}{b_2+\cfrac{a_3x^2}{b_3+\ddots}}}dx$$ Can you use closed form?

Short title.
 
11:40 AM
0
Q: If $A=XX^*$ then all eigenvalues of $A^{(-1)}$ are positive?

H.SLet $A\in M_n $, we define $A^{(−1)} = [a^{−1}_{i j} ]$. Suppose $A=XX^*$ and $A$ be positive semidefinite. Why are, all eigenvalues of $A^{(-1)}$ positive?

Short question.
0
Q: How to calculate $\sqrt{1 + \sqrt{2 + \sqrt{4 + \sqrt{8 + ...}}}}$.

T. GoldenLet $s = \sqrt{1 + \sqrt{2 + \sqrt{4 + \sqrt{8 + ...}}}}$. $$st = t\sqrt{1 + \sqrt{2 + \sqrt{4 + \sqrt{8 + ...}}}} = \sqrt{t^{2} + \sqrt{2t^{4} + \sqrt{4t^{8} + \sqrt{8t^{16} + ...}}}}$$ Let $2t^{4} = t^{2}$: $$2t^{2} = 1$$ $$t^{2} = \frac{1}{2}$$ $$t = \frac{1}{\sqrt{2}}$$ $$\frac{s}{\sqrt{2}...

Tagged proof-verification.
0
Q: find a limit depending on parametres a,b in N

sambo14Limit $ a,b\in \mathbb {N} \ lim_{x\to\infty} ((n+1)^k + (-n)^l) /((n-1)^k -n^l)$ I am really stuck, I do not know where to start. I tried to find different case when k is bigger than l or when k is smaller, but it seems it is getting me nowhere. Thanks for help

Consider replacing (analysis) with a more specific tag for the relevant branch of analysis. (autocomment)Normal Human 21 secs ago
0
Q: Ways of checking a correct ordinary point?

JONATHONDOEINGIf you have derived a power series expansion for an ordinary point, is there any way to check whether your answer is correct?

Short question.
0
Q: How can I make this proof to make sense?

javalolProve that any (simple, undirected) graph G = (V, E), with m = |E| edges has chromatic number χ(G) ≤ √2m + 1.

Short question.
0
Q: Simplifying a 2- form

user290605In the link here http://www.ulb.ac.be/sciences/ptm/pmif/Rencontres/specgeom.pdf, p.4, it says that, given a fundamental 2-from $\mathcal{K}$ $$\mathcal{K}=\frac{i}{2\pi}g_{i\bar{j}}dz^i\wedge d\bar{z}^{\bar{j}}$$, a manifold is said to be Kahler if this form is closed, i.e., $$d\mathcal{K} = i\pa...

Short title.
0
Q: If $P(X<Y)=P(X<g(Y))$ then what could be the form of $g$?

user23510Let $X$ and $Y$ are two continuous random variable and $$P(X<Y)=P(X<g(Y)),$$ for some convex function $g$. Then is it true that $g$ is a linear function?

Short question.
0
Q: looking for a contradictory proof

math101Let $f \in L^p$, where $p \ge 1$, then $|f(x)| < \infty$ almost everywhere. Does anyone know how to prove this by contradition?

Short question.
0
Q: Probability function of two independent uniform random variables

KageIf X and Y are both random variables with continuous uniform distribution with bounds [0,1], what is the distribution of XY? X~U[0,1] Y~U[0,1] XY~???

Short question.
0
Q: Finding conditional distribution in multinormal case

Stella ParkExample I'm trying to understand this example and can't figure out how it partition the covariance matrix. In this case, what is {Sigma_22}inverse ? Thank you for your help.

Short question.
0
Q: Combs and Perms

user29060911 letters are selected with replacement from the word ‘BRACED’. i) How many different combinations of letters are possible? ii) How many distinguishable permutations of ‘ABRACADABRA’ are there? I do not understand how 11 letters are selected, with replacement, from a 6-letter word. However, her...

Short title.
0
Q: Distribution of quantifiers.

dwiet85Is the statement ∃x(S(x)∧∀y(F(y)→A(x,y))), equivalent to ∃x∀y(S(x)∧F(y)→A(x,y))? Since from ∃x∀y(S(x)∧F(y)→A(x,y)), you can distribute ∀y across the statement, obtaining, ∃x(∀yS(x)∧∀yF(y)→∀yA(x,y)). Since x and y are not dependent, we obtain ∃x(S(x)∧∀yF(y)→∀yA(x,y)), which can be shortened to ∃...

Short title.
 
12:07 PM
-2
Q: Request for a review to re-open a question after an edit

EigirI asked a question that got put on-hold and I agree with the reason. I edited the question, and looking on how fast it was put on-hold, I guessed that a new review would be equally fast. I know, I can be pretty naive some times. :) One thing is the time it is spent in the reopen queue, but what ...

 
0
Q: Interesting Recursion Question

user290610I am taking a course in algebra and this problem was on my problem set, and I had no idea how to solve it. Suppose we have a sequence $s_n$ such that $5*s_{n+1}-s_{n}-3*s_{n}s_{n+1}=1$ for $1 \leq n \leq 42$ and $s_1=s_{43}$. What is $s_1+s_2+ \ldots + s_{42}$?

Welcome to Math.SE, user290610. Words such as interesting, question do not add information to titles. Please edit the title so that it better describes the specifics of your question. Do not hesitate to make it longer or include a formula if needed. More tips here. (autocomment)Normal Human 21 secs ago
0
Q: how to find local or global minimizer

Elie Absenter image description here I have solved the first two parts, but i didn't know who to find the rest, can someone help ?

Short question.
0
Q: Tricky infinite integral

MarkHow to prove that $$\int_0^\infty \frac{x\sin x}{x^2+1}=\frac{\pi}{2e}$$ I've tried several basic approaches like substitution and IBP but can't move forward.

Short title. Title contains tricky. Short question.
 
0
Q: One correct answer and one hack, that does work better for me - which to accept?

JC_CLI asked a question here and so far it has two answers. The first one by Pedro M Duarte, which I feel is the correct answer, and the second one, by Nader Hisham, which does ignore the index as a way of sorting, but rather "pollutes" my data, with some unrelated sorting info. But, I can easily ge...

 
12:32 PM
0
Q: Prove that ∀ n ∈ {n | n ∈ ℕ ∧ n > 7}: n = x*3 + y*5

CrushedPixelHow would I prove that for all elements of a set {n | n ∈ ℕ ∧ n > 7} (all natural numbers greater than 7), there are multiples of 3 and 5 which, when added up, are equal to that element of the set? A := {n | n ∈ ℕ ∧ n > 7} ∀ n ∈ A: n = x*3 + y*5 I do understand why this is true: 8 = 1*3 + 1*...

Title contains 5.
 
1:04 PM
0
Q: Why find triangles in graphs?

SummerGiven a graph, there are numerous algorithms out there to find a set of 3 vertices such that a triangle exists, but why is this topic studied? What are the applications? The advantages? The problems encountered?

Short title.
 
0
Q: Posting questions about file directory structure

Sam SwiftI wanted to know about the naming conventions and structure a file system for a PHP site should take, but for this I am unaware of the best SO community to post this on. Could someone direct me to the right community to ask this question?

 
0
Q: Center of Mass in 2 triangle

Alpay NamaziI've got a question. As you know if we connect the dots on the medians , it makes another triangle inside the main triangle, so here's my questions : 1. Are the line flashes drew rightly? [As the picture] 2. If we make the second triangle (Red one) bigger , does it have similarity with bigger o...

Short title.
0
Q: Bijection between groups

testLet $G=\{A\in M_{2x2}(\Bbb{F})\colon \det A\not=0\}$ be a group and $Z(G)\subset G$. I am working on a project(lin.alg.) and I am curious if there is a mapping(bijectively) that maps $G$ to $Alt(5)$? Is this a known problem?

Short title.
 
1:44 PM
0
Q: Mobius Transformation under w for unit disc and unit circle

JaydenWhat happens to the unit circle and unit disc under the Mobius Transformation w = (z+i)/(z-i)? I don't know how to go about setting this out...i.e. how do i represent the disc as a complex equation?

Consider adding a tag for a broader subject area to which the question belongs. Some of these tags might fit. (autocomment)Normal Human 21 secs ago
0
Q: limit of integration

MBdrKnowing that the function $$\frac{1}{\sqrt (1-x^2)}\ $$ is defined only for -1 < x <1 Are the limits of integration below allowed? $$\int_{-1}^1 \frac{1}{\sqrt (1-x^2)} \,dx= \pi$$ PS: As stated above, I can calculate the result as pi, I'm just questioning the reasoning behind using -1 and ...

Short title.
0
Q: Elementary Proof of Positive + Negative Decomposition of Self-Adjoint Operators on Hilbert Spaces

dmtrGiven a compact, self-adjoint operator $T$ on a Hilbert space $\mathcal{H}$ I know that we can write it as $T=T_++T_-$ where $T_+$ and $T_-$ are, respectively, positive and negative and $T_+T_-=0$. However, all of the proofs that I can find of this fact seem to rely on using continuous functional...

Title contains elementary.
0
Q: Interpretation of PCA

JohnSmithI am wondering if there is a practical interpretation of a principal component analysis: Consider you have a data matrix $X\in\mathbb{R}^{N\times p}$ and you perform a principal component analysis where you typically receive certain directions $v_1,...,v_q$, $q<p$, in $\mathbb{R}^N$ that explain ...

Short title.
 
00:00 - 14:0014:00 - 00:00

« first day (14 days earlier)      last day (560 days later) »